Gönderen Konu: Uluslararası Matematik Olimpiyatı 2004 Soru 4  (Okunma sayısı 3028 defa)

Çevrimdışı ERhan ERdoğan

  • G.O Genel Moderator
  • Geo-Maniac
  • ********
  • İleti: 1424
  • Karma: +12/-0
Uluslararası Matematik Olimpiyatı 2004 Soru 4
« : Haziran 05, 2014, 08:56:27 ös »
$n\geq 3$ bir tam sayı olmak üzere; $t_1,t_2,\dots, t_n$ pozitif gerçel sayıları $$n^2+1 > (t_1+t_2+\dots+t_n)\left(\dfrac 1{t_1} + \dfrac 1{t_2} + \dots + \dfrac 1{t_n} \right)$$ koşulunu sağlasın. $1\leq i < j < k \leq n$ koşulunu sağlayan her $i$, $j$, $k$ sayıları için $t_i$, $t_j$, $t_k$ sayılarının bir üçgenin kenarları olduğunu gösteriniz.

Çevrimdışı Hüseyin Yiğit EMEKÇİ

  • Geo-Maniac
  • ********
  • İleti: 602
  • Karma: +2/-0
Ynt: Uluslararası Matematik Olimpiyatı 2004 Soru 4
« Yanıtla #1 : Aralık 31, 2023, 07:50:28 ös »
Genelliği bozmadan $t_n\geq t_{n-1}\geq \cdots \geq t_1$ olsun.
Soruda bu eşitsizliği sağlayan $t_i$ lerden herhangi farklı üçünün bir üçgen oluşturduğu ispatlamamız isteniyor. Aksini varsayalım, $t_n\geq t_1+t_2$ olsun. Buna göre $t_n\geq \dfrac{2t_n}{5}+\dfrac{3\left(t_1+t_2\right)}{5}$ olduğunu söyleyebiliriz.
Dolayısıyla
$$LHS=\left(t_1+t_2+\cdots+t_n\right)\left(\dfrac{1}{t_1}+\dfrac{1}{t_2}+\cdots+\dfrac{1}{t_n}\right)\geq \left(t_1+t_2+\cdots+t_{n-1}+\dfrac{2t_n}{5}+\dfrac{3\left(t_1+t_2\right)}{5}\right)\left(\dfrac{1}{t_1}+\dfrac{1}{t_2}+\cdots+\dfrac{1}{t_n}\right)$$
$$=\left(\dfrac{8t_1}{5}+\dfrac{8t_2}{5}+t_3+\cdots+t_{n-1}+\dfrac{2t_n}{5}\right)\left(\dfrac{1}{t_1}+\dfrac{1}{t_2}+\cdots+\dfrac{1}{t_n}\right)\overbrace{\geq}^{Cauchy} \left(n+\sqrt{10}-3\right)^2$$
Sondaki eşitsizliği gösterelim
$$=\left(\dfrac{8t_1}{5}+\dfrac{8t_2}{5}+t_3+\cdots+t_{n-1}+\dfrac{2t_n}{5}\right)\left(\dfrac{1}{t_1}+\dfrac{1}{t_2}+\cdots+\dfrac{1}{t_n}\right)$$
$$\geq \left(\sqrt{\dfrac{8t_1}{5}}.\sqrt{\dfrac{1}{t_1}}+\sqrt{\dfrac{8t_2}{5}}.\sqrt{\dfrac{1}{t_2}}+\sqrt{t_3}.\sqrt{\dfrac{1}{t_3}}+\sqrt{t_4}.\sqrt{\dfrac{1}{t_4}}+\cdots+\sqrt{t_{n-1}}.\sqrt{\dfrac{1}{t_{n-1}}}+\sqrt{\dfrac{2t_n}{5}}.\sqrt{\dfrac{1}{t_n}}\right)^2$$
$$=\left(\sqrt{\dfrac{8}{5}}+\sqrt{\dfrac{8}{5}}+\overbrace{1+1+\cdots+1}^{n-3}+\sqrt{\dfrac{2}{5}}\right)^2=\left(n-3+\sqrt{10}\right)^2$$
elde edilir.
Sonuç olarak $t_n\geq t_1+t_2$ durumunda
$$LHS\geq \left(n+\sqrt{10}-3\right)^2\geq n^2+1$$
olduğunu gösterirsek çelişkiyi elde etmiş olacağız. Gösterelim
$$\left(n+\sqrt{10}-3\right)^2=n^2+2n\left(\sqrt{10}-3\right)+19-6\sqrt{10}\geq n^2+1$$
$$\Rightarrow 2n\left(\sqrt{10}-3\right)+18-6\sqrt{10}\geq 0$$
Sol tarafın $\left(2n-6\right)\left(\sqrt{10}-3\right)\geq 0 \Rightarrow n\geq 3$ olması ile biter.
Özetle çelişkiyi elde ettik. Çelişkimiz ise şudur :
$t_n\geq t_1+t_1$ durumunda $LHS\geq n^2+1$ elde edilir. Bundan dolayı $t_n< t_1+t_2$ olmalıdır dolayısıyla üçgen oluşturmaları gerekir. Boylelikle ispat tamamlanır.

Problemin ispatı, genelliği bozmadan kısmında $\max{t_1}=t_n$ verilerek de gösterilebilirdi. Ayrıca her üçgen oluşturan $t_i$ ler eşitsizliği sağlamak zorunda değillerdir ama tersi doğrudur.
« Son Düzenleme: Ocak 25, 2024, 01:58:20 ös Gönderen: Hüseyin Yiğit EMEKÇİ »
''Uzman, çok dar bir alanda yapılabilecek tüm hataları yapmış kişidir.''   ~Niels Bohr

Çevrimdışı Hüseyin Yiğit EMEKÇİ

  • Geo-Maniac
  • ********
  • İleti: 602
  • Karma: +2/-0
''Uzman, çok dar bir alanda yapılabilecek tüm hataları yapmış kişidir.''   ~Niels Bohr

 


Sitemap 1 2 3 4 5 6 7 8 9 10 11 12 13 14 15 16 17 18 19 20 21 22 23 24 25 26 27 28 29 30 31 32 33 34 35 36 37 
SimplePortal 2.3.3 © 2008-2010, SimplePortal